So–called environmentalists have argued that the proposed Golden Lake Development would interfere with bird–migration...

AndrewArabie on November 28, 2022

Answer Choice and explanation

The explanation is treating this as a Strengthen with Necessary question but its a Strengthen with Sufficient so the explanation doesn't explain to me how A is correct.

Reply
Create a free account to read and take part in forum discussions.

Already have an account? log in

Emil-Kunkin on December 8, 2022

Hi, I think A is both a necessary and sufficient assumption, that said, let's look at its negation. If it were the case that the group had indeed opposed every proposal because they would have done harm to the environment, then the authors argument falls apart. There is no proof they are just anti progress if they did have a legitimate environmental justification for their opposition.